Download Solutions - Cornell Math

Survey
yes no Was this document useful for you?
   Thank you for your participation!

* Your assessment is very important for improving the work of artificial intelligence, which forms the content of this project

Document related concepts
no text concepts found
Transcript
Honors Introduction to Analysis I
Homework III
Solution
February 15, 2009
Problem 1 Let x be a real number. Show that there exists a Cauchy sequence of rationals {xn } representing x
such that xn ≤ xn+1 for every n.
Solution. We will build two sequences, one increasing, and one decreasing, proving a more general result.
If x ∈ Q, consider the constant sequence xn = x, for any n.
If x ∈
/ Q, choose rational numbers y1 and z1 such that y1 < x < z1 (we can do this by the completeness of the
1
reals and the density of rationals). Consider m1 = y1 +z
2 , which is rational. If x > m1 , define y2 = m1 and z2 = z1 ,
and if x < m1 , define y2 = y1 and z2 = m1 .
2
Next, we iterate, considering m2 = y2 +z
2 , and take y3 = m2 and z3 = z2 if m2 < x or y3 = y2 and z3 = m2 if
m2 > x. Similarly we get y4 , z4 from y3 , z3 and so on.
We repeat the process of dividing infinitely often, and we will obtain two sequences: {yk } (increasing) and {zk }
−y1
(decreasing), so that yk ≤ x ≤ zk and zk − yk = z21k−1
.
−y1
Both are equivalent Cauchy sequences: given 1/n, take m large enough so that z21m−1
< 1/n. Then 0 ≤ zm −ym <
1/n and for k ≥ m, [yk , zk ] ⊂ [ym , zm ], so |yj − yk | < 1/n, |zj − zk | < 1/n and |yj − zk | < 1/n, for j, k ≥ m. Since
the sequences are equivalent Cauchy, they will have a common limit, call it y.
But since yk ≤ x ≤ zk , then taking the limit will yeald: y ≤ x ≤ y, so x is their common limit. So we have build
an increasing sequence {yn } and a decreasing sequence {zn }, both representing x.
Problem 2 Prove that there are an infinite number of rational numbers in between any two distinct real numbers.
Solution. Let x < y be the two real numbers. By the Archimedean property, there is an m ∈ N such that
y − x > 1/m.
By the density of the rationals, there is a rational r1 such that |r1 − x| < 1/(3m) and r1 > x (we can make sure
r1 > x by an argument similar to the previous problem) and s1 such that s1 < x and |y − s1 | < 1/(3m). By the
first condition, we know that x < r1 < s1 < y. But between any two rationals (in particular for r1 and s1 ), there
are infinitely many rationals (we can consider a sequence {rk }, defined such that rk is the middle of [rk+1 , s1 ]).
Hence, as x < r1 < s1 < y, there are infinitely many rationals between x and y.
Problem 3 Prove the following identities from the field axioms:
ax2 + bx + x = a(x + b/2a)2 + c − b2 /(4a)
n X
n k n−k
n
(x + y) =
x y
k
k=0
Solution. One can deduce the properties used from the steps.
1
•
2a
b2
b2
bx + c +
−
2a
4a 4a
2
b
b
b2
= ax2 + a2x +
+c−
2a 4a
4a
2
b2
b
b
= ax2 + a2x + a 2 + c −
2a
4a 4a
2
b
b2
b
= a x2 + 2x + 2 + c −
2a 4a
4a
2
2
b
b
=a x+
+c−
2a
4a
ax2 + bx + c = ax2 +
• We will use induction on n ≥ 0. For n = 0, we have (x + y)0 = 1 =
0 0 0
x y = 1 · 1 · 1 = 1.
0
Asssume it is true for n and we prove it for n + 1.
n+1
(x + y)
n X
n
n
= (x + y) (x + y) =
n X
n
k
!
k n−k
x y
(x + y)
k=0
n X
n k n−k+1
x y
k
k
k=0
k=0
n n X
X
n
n k n−k+1
n+1
n+1
k n−k+1
=x
+y
+
x y
+
x y
k−1
k
k=1
k=1
n X
n
n
n+1
n+1
=x
+y
+
+
xk y n−k+1
k−1
k
k=1
n X
n + 1 k n−k+1
x y
= xn+1 + y n+1 +
k
=
xk+1 y n−k +
k=1
=
n+1
X
k=0
n + 1 k n+1−k
x y
k
We have used the identity
n
n
n+1
+
=
.
k−1
k
k
Problem 4 Prove the following identities from the ordered field axioms:
(x2 + y 2 )/x2 ≥ 1, x 6= 0
2xy ≤ x2 + y 2
x/y > x if x > 0 and 0 < y < 1
Solution.
•
(x2 + y 2 )/x2 = x2 · (x2 )−1 + y 2 · (x2 )−1 = 1 + y 2 · (x2 )−1 ≥ 1 + 0 = 1
Since y 2 > 0 and x2 > 0 (hence (x2 )−1 > 0) and multiplication of positive numbers is positive.
• As (x − y)2 ≥ 0 (square of any real is positive), we get: x2 − 2xy + y 2 ≥ 0. By adding on both sides 2xy, we
don’t change the direction of the inequality, so
(x2 − 2xy + y 2 ) + 2xy ≥ 2xy ⇔ x2 + y 2 ≥ 2xy
2
• As 0 < y < 1, then 0 < 1 < y −1 , hence, as multiplication with a postive number x preserves the direction of
the inequality, x · 1 < x · y −1 , so x < xy
Problem 5 Prove that every real number x has a unique real cube root.
Solution.
Existence We apply the “divide and conquer” method. We can just analyze the case when x ≥ 0, since for the
case x ≤ 0, we deal in the same way, with the difference√that the inequalities will go in the opposite direction.
First we find the numbers y1 and z1 such that y = 3 x lies between them. If x > 1 then x3 = x + x(x2 − 1) =
x + x(x − 1)(x + 1) > x since x(x − 1)(x + 1) > 0. From 1 < x < x3 , we expect 1 < y < x, so set y1 = 1 and z1 = x.
(Note that if x < −1, then x3 < x, so we will set y1 = x and z1 = −1). Also if 0 < x < 1, we take y1 = x and
z1 = x (if −1 < x < 0, we switch them: y1 = x and z1 = x). If x = ±1 then take y = ±1.
1
So we have y13 ≤ x ≤ z13 . Now we consider the midpoint m1 = y1 +z
2 . We choose the next interval y2 to z2 to be
3
either [y1 , m1 ] or [m1 , z1 ] in the following way: if m1 > x, take y2 = y1 and z2 = y1 ; if m31 < x, take y2 = m1 and
z2 = z1 ; if m31 = x, then set y = m1 . We will have y23 ≤ x ≤ z 3 .
3
3
2
We iterate, considering m2 = y2 +z
2 , and take y3 = m2 and z3 = z2 if m2 < x or y3 = y2 and z3 = m2 if m2 > x
3
or y = m2 if m2 = x. Similarly we get y4 , z4 from y3 , z3 and so on. √
We repeat the process of dividing infinitely often (unless we hit 3 x), and we will obtain two sequences: {yk }
−y1
(increasing) and {zk } (decreasing), so that yk3 ≤ x ≤ zk3 and zk − yk = z21k−1
.
−y1
< 1/n. Then 0 ≤ zm −ym <
Both are equivalent Cauchy sequences: given 1/n, take m large enough so that z21m−1
1/n and for k ≥ m, [yk , zk ] ⊂ [ym , zm ], so |yj − yk | < 1/n, |zj − zk | < 1/n and |yj − zk | < 1/n, for j, k ≥ m.
Since the sequences are equivalent Cauchy, they will have a common limit, call it y. Since yk3 ≤ x ≤ zk3 , passing
to the limit gives: y 3 ≤ x ≤ y 3 , so y 3 = x, hence x has a cubic root.
Uniqueness Assume there are two distinct cubic roots z 3 = y 3 = x. Then
0 = z 3 − y 3 = (z − y)(z 2 + zy + y 2 ) = (z − y)[(z + y/2)2 + 3/4y 2 ]
But (z + y/2)2 + 3/4y 2 > 0, since if both terms would be 0, then y = z = 0 (contradictinv z, y distinct), hence
z − y = 0, so z = y proving uniqueness.
Problem 6 Let {xn } be a sequence of real numbers such that |xn | ≤ 1/2n , and set yn = x1 + x2 + . . . + xn . Show
that the sequence {yn } converges.
Solution. We will show that {yn } is Cauchy. We observe that, for any j > k:
j
j
j
X
X
X
1
1
1
1
1
|yj − yk | = xn ≤
|xn | ≤
=
1
+
+
.
.
.
+
= k+1
n
k+1
j−k−1
2
2
2
2
2
n=k+1
n=k+1
n=k+1
1
−
2j−k
1
2
1
=
1
1
1
− k ≤ j
j
2
2
2
This means that for any n ∈ N, there is m such that 2m is the closest power of 2 to n (and 2m > n), and for any
j ≥ k ≥ m, we have
1
1
1
|yj − yk | ≤ j ≤ m ≤
2
2
n
Hence {yn } is indeed Cauchy, so by the completeness of the reals, it converges.
Problem 7 Prove that the irrational numbers are dense in R.
Solution. We have to prove that given any real number x and error 1/n, we can find an irrational number y such
that |x − y| ≤ 1/n.
Given any real x, there is a Cauchy sequence of rationals {xn } that represents it. So given any n, there is an m
such that |xj − xk | ≤ 1/(2n), for j, k ≥ m.
3
√
√
Define yn = xn + 2 ∈ R − Q (assume it is equal to q ∈ Q, then 2 = n(q
− xn ) ∈ Q, contradiction). This new
√
2
1
(it exists by the Archimedean
sequence is Cauchy, because, given any n, there is an m0 (chosen such that m0 < 2n
property).) such that for j, k ≥ m0 :
√
√ 1 1
1 √
1 1
1 √ 1
1
1
1
|yk −yj | = |(xk −xj )+ 2(1/k−1/j)| ≤ |xj −xk |+ 2 − ≤
+ 2 max
,
≤
+ 2
<
+
=
j
k
2n
k j
2n
m0
2n 2n
n
√
It is also equivalent to {xn }, because, for any n, there is m1 > 2n, such that for k ≥ m1 :
√ √
2
2
1
|yk − xk | = <
≤
k m1
n
Hence they both represent x, so the limit of {yk } is also x. Hence, given any error 1/n, since yn → x, we can
find an element of the sequence yt ∈ R − Q such that |x − yt | < 1/n. Hence the irrationals are dense in R.
Problem 8 Let {pk } and {qk } be the sequences considered
in the lecture: p1 = q1 = 1, pk+1 = pk + 2qk , qk+1 =
√
pk + qk . Let ek be the error term of approximation of 2 by the fraction pk /qk . Estimate ek+1 in terms of ek . What
is roughly the degree of accuracy of the k th approximant? Show that ek tends to 0 as k goes to infinity.
Solution. First let’s notice that pk and qk are positive numbers, for any k. Then for k ≥ 2:
1=
pk−1 + qk−1
pk−1 + 2qk−1
pk
2pk−1 + 2qk−1
≤
=
≤
=2
pk−1 + qk−1
pk−1 + qk−1
qk
pk−1 + qk−1
So ak := pqkk ∈ [1, 2], so we get that ak1+1 ∈ [ 13 , 21 ] (*).
Also notice that:
pk+1
pk + 2qk
1
1
ak+1 =
=1+
=
= 1 + pk
qk+1
pk + qk
1 + ak
qk + 1
√
We define ek = pqkk − 2 and we get a relation between ek+1 in terms of ek :
pk+1 √ √ √
√ √
√ √ 1
ek+1
qk+1 − 2 1 + 1+ak − 2 ak + 1 + 1 − 2ak − 2 (1 − 2)(ak − 2) 1 − 2 √
√
√
= pk √ = =
=
=
(ak − 2)(ak + 1) (ak − 2)(ak + 1) ak + 1 q − 2 ek
ak − 2
k
√ √
√
√
1− 2 |1− 2|
So ek+1
=
≤
(by (*)). Since 1 < 2 < 4 it means that 1 < 2 < 2, and as 2 > 1/4, then 2 − 1 > 1/2
ek
ak +1
2
√
so |1 − 2| < 12 . We therefore get that:
ek+1
1
1
≤ ⇒ ek+1 ≤ ek
ek
4
4
1
This means that ek ≤ 4k−1
e1 , so the degree of accuracy of the k th approximant is of the order of 1/4k .
1
The sequence bk = 4k−1 e1 goes to 0 as k → ∞, and as 0 ≤ ek ≤ bk , by the squeeze criterion of converging
sequences, we have that lim ek = 0 (alternatively one can prove that {ek } is Cauchy).
k→∞
Problem 9 Write your version of the proof of completeness of the real numbers.
Solution. This is just Theorem 2.3.1 - the problem has been checked, but not counted toward the points, since
it’s just an exarcise meant for you to go over the proof on your own.
4
Related documents